The product 8 and the square of a number decreased by 5 is 67. Find the number.

Answers

Answer 1

Answer:

3 or -3

Explanation:

Let's call the unknown number x. The square of this number is x². The product of 8 and the square of this number is 8x². Finally, it is decreased by 5, so 8x² - 5 and it is equal to 67, then the equation that represents the statement is:

8x² - 5 = 67

Now, we can solve the equation for x. Add 5 to both sides

8x² - 5 + 5 = 67 + 5

8x² = 72

Divide both sides by 8

8x²/8 = 72/8

x² = 9

Find the square root of both sides

x = √9

x = 3 or x = -3

Therefore, the number is 3 or -3


Related Questions

I need help with a math question. I linked it below

Answers

[tex]\begin{gathered} \frac{b}{55}+8>6 \\ \frac{b}{55}>-2 \\ b>-110 \end{gathered}[/tex]

1) We can fill in the gaps, this way since we can write the following when we translate into mathematical language:

[tex]\begin{gathered} \frac{b}{55}+8>6 \\ \frac{b}{55}>-8+6 \\ \frac{b}{55}>-2 \\ 55\cdot\frac{b}{55}>-2\cdot55 \\ b>-110 \end{gathered}[/tex]

Note that we could do it in two steps. Subtracting and then multiplying and dividing

Which tool would be best to solve this problem?Pythagorean TheoremTriangle Angle Sum TheoremTangent RatioSine RatioCosine RatioUse that tool to solve for x. Show all work on the sketchpad or on your paper.

Answers

To get the value of x,

We use triangle sum theorem:

Triangle Sum Theorem:

The sum of all angles in a triangle is equal to 180 degrees.

In the triangle

We have 90 degree, 22 degree and one x

So,

x + 90 + 22 =180

x + 112 = 180

x = 180-112

x = 68 degree

Answer: x = 68

Write a quadratic equationwith vertex (3,-6) and otherpoint (-7,14). Solve for a!

Answers

We have to find the parameter a of a quadratic equation knowing the following

• The vertex is (3,-6).

,

• A random point is (-7,14).

Based on the given information, we have the following

[tex]\begin{gathered} h=3 \\ k=-6 \\ x=-7 \\ y=14 \end{gathered}[/tex]

The vertex form of a quadratic equation is

[tex]y=a(x-h)^2+k[/tex]

Replacing all the givens, we have

[tex]14=a(-7-3)^2-6[/tex]

Now, we solve for a

[tex]\begin{gathered} 14=a(-10)^2-6 \\ 14=a(100)-6 \\ 14+6=100a \\ 100a=20 \\ a=\frac{20}{100}=\frac{1}{5} \end{gathered}[/tex]Therefore, a is equal to 1/5.

find 2x:3y if x:y = 2:5

Answers

4 : 15

Explanation:[tex]\begin{gathered} \text{x : y = 2: 5} \\ \frac{x}{y}\text{ = }\frac{2}{5} \\ \\ 2x\text{ : 3y = ?} \end{gathered}[/tex][tex]\begin{gathered} 2x\colon\text{ 3y = }\frac{2x}{3y} \\ 2x\colon3y\text{ = }\frac{2}{3}\times\frac{x}{y} \end{gathered}[/tex][tex]\begin{gathered} \text{substitute for x/y in 2x:3y} \\ \frac{2}{3}\times\frac{x}{y}\text{ =}\frac{2}{3}\times\frac{2}{5} \\ =\text{ }\frac{4}{15} \\ \\ \text{Hence, 2x:3y = }\frac{4}{15} \\ or \\ 4\colon15 \end{gathered}[/tex]

(4xy³y⁴)(5x²y) expand and simplify

Answers

In this case, we'll have to carry out several steps to find the solution.

Step 01:

Data

(4xy³y⁴)(5x²y)

Step 02:

[tex](4xy^3y^4)(5x^2y)=(4xy^7)(5x^2y)=20x^3y^8^{}[/tex]

This is the solution.

[tex]20x^3y^8^{}[/tex]

What's the equation of the axis of symmetry of g(x)=x^{2}+4 x+3?A) x=0B) x=-2C) x=2D) x=3

Answers

Given a quadratic equation of the form:

[tex]f(x)=ax^2+bx+c[/tex]

The equation of the axis of symmetry is obtained using the formula:

[tex]x=-\frac{b}{2a}[/tex]

From the given quadratic equation:

[tex]\begin{gathered} g\mleft(x\mright)=x^2+4x+3 \\ a=1 \\ b=4 \end{gathered}[/tex]

Therefore, the equation of the axis of symmetry of g(x) is:

[tex]\begin{gathered} x=-\frac{4}{2\times1} \\ x=-2 \end{gathered}[/tex]

The correct option is B.

Identify the following forms of factoring with the correct method of solving

Answers

Given:

There are given the equation:

[tex]90x^3-20x[/tex]

Explanation:

To find the factor of the given equation, first, we need to take a common variable from the given equation:

[tex]90x^3-20x=x(90x^2-20)[/tex]

Then,

[tex]\begin{gathered} 90x^3-20x=x(90x^2-20) \\ =10x(9x^2-2) \end{gathered}[/tex]

Final answer:

Hence, the factor of the given equation is shown below:

[tex]\begin{equation*} 10x(9x^2-2) \end{equation*}[/tex]

If y = (x/x+1)5, then dy/dx

Answers

The value of dy/dx is 5x^4 / (x + 1)^6.

What is the derivative?

A function's sensitivity to change with respect to a change in its argument is measured by the derivative of a function of a real variable.

The given function is y = (x / (x + 1))^5

Taking derivative on both sides,

dy/dx = d/dx (x / (x + 1))^5)

Using chain rule,

dy/dx = 5(x /  x + 1)^4 x d/dx (x / x + 1)

Using the quotient rule of derivative,

d/dx (x / x + 1) = 1 / (x + 1)^2

So,

dy/dx = 5(x / x+1)^4 x (1 / (x + 1)^2)

dy/dx = 5x^4 / (x + 1)^6

Therefore, the derivative of the given function is, dy/dx = 5x^4 / (x + 1)^6.

To know more about the derivative of a function, click on the link

https://brainly.com/question/12047216

#SPJ13

1. Alice made the conjecture below.(a + b)2 = a + b2OWhich values of a and b are not counterexamples to the conjecture?a = 1, b = 1a = 0, b = 1aa = -1, b = 1a = -1, b = 2

Answers

the expression is

(a+b)^2=a+b^2

substitute a=1 and b=1

(1+1)^2 = 1+1^2

4=2

that is not possible. so these are the values of a and b that is not counterpart example to the conjecture.

now substitute a=0, b=1

(0+1)^2 = 0+1^2

1=1

so this is true for the above expression.

now for a=-1, b=1

(-1+1)^2 = -1+1^2

0=0

this is true.

now for a=-1, b=2

(-1+2)^2 = -1+2^2

1=3

that is not possible

so a=1, b=1 and a=1,b=2 is the values that not counterpart example to the conjecture.

Choose the algebraic description that maps ΔABC onto ΔA′B′C′ in the given figure.Question 7 options:A) (x, y) → (x + 4, y + 8)B) (x, y) → (x + 8, y + 4)C) (x, y) → (x – 4, y – 8)D) (x, y) → (x + – 8, y – 4)

Answers

Step 1

Given the triangle, ABC translated to A'B'C'

Required to find the algebraic description that maps triangle ABC and A'B'C'

Step 2

The coordinates of points A, B,C are in the form ( x,y)

Hence

[tex]\begin{gathered} A\text{ has a coordinate of ( -3,-2)} \\ B\text{ has a coordinate of (-6,-5)} \\ C\text{ has a coordinate of (-1,-4)} \end{gathered}[/tex]

Step 3

Find the algebraic description that maps triangle ABS TO A'B'C'

[tex]\begin{gathered} A^{\prime}\text{ has a coordinate of (5,2)} \\ B^{\prime}\text{ has a coordinate of ( 2,-1)} \\ C^{\prime}\text{ has a coordinate of ( 7, 0)} \end{gathered}[/tex]

The algebraic description is found using the following;

[tex]\begin{gathered} (A^{\prime}-A^{})=(x^{\prime}-x,\text{ y'-y)} \\ OR \\ (B^{\prime}-B)=(x^{\prime}-x,\text{ y'-y)} \\ OR \\ (C^{\prime}-C)=(x^{\prime}-x,\text{ y'-y)} \end{gathered}[/tex]

Hence,

[tex]\begin{gathered} =\text{ ( 5-(-3)), (2-(-2))} \\ =(8,4) \\ \text{Hence the algebraic description from triangle ABC to A'B'C' will be } \\ =(x,y)\Rightarrow(x\text{ + 8, y+4)} \end{gathered}[/tex]

Hence the answer is option B

Elsie is moving to iowa city iowa, with her three-year-old daughter. The table shows the results of a family budget estimator for iowa City for Elsie and her daughter."If Elsie earns $45,000 per year at her new job, can she stay on budget in lowa City? A. Yes, because she can easily afford $4020 per month.B. Yes, because she will not actually need all the items that the family budget estimator includes. C. No, because she will only make $3750 per month before taxes are taken out. D.No, because she will not be able to find housing as low as $853 per month?

Answers

In order to determine what is the correct statement, calculate the amount of money Elsie can spend per month, based on her earnings per year.

Divide 45,000 by 12:

45,000/12 = 3,750

You can notice that the amount of money Elsie can spend per month is lower than the total expenses shown in the table.

Hence, the correct statement is:

C. No, because she will only make $3750 per month before taxes are taken out.

Write the equation of the line when the slope is 1/5 and the y-intercept is 13.

Answers

Given:

• Slope, m = 1/5

,

• y-intercept = 13

Let's write the equation of the line.

To write the equation of the line, apply the slope-intercept equation of a line:

[tex]y=mx+b[/tex]

Where:

m is the slope

b is the y-intercept.

Thus, we have:

m = 1/5

b = 13

Plug in the values in the equation:

[tex]y=\frac{1}{5}x+13[/tex]

Therefore, the equation of the line is:

[tex]y=\frac{1}{5}x+13[/tex]

Solve the following simultaneous equation using the inverse matrix method

3x + 4y = 18
4x - y = 5

Answers

The values of x and y obtained by inverse matrix method are 2 and 3 respectively

What is an inverse matrix?

An inverse of a matrix, A, is a matrix [tex] A^{-1}[/tex], that multiplies matrix A to give an identity matrix.

The inverse matrix method involves defining and making use of a coefficient matrix, A, a variable matrix, X, and a constant matrix, B, which are obtained from the system of equations as follows;

A·X = B
The system of equations is presented as follows;

3·x + 4·y = 18

4·x - y = 5

From the above system of equations, we have:

[tex]The \ coefficient \ matrix, \ A = \begin{bmatrix} 3&4 \\ 4& -1 \end{bmatrix}[/tex]

[tex]The \ variable \ matrix, \ X = \begin{bmatrix} x \\ y \end{bmatrix}[/tex]

[tex]The \ constant \ matrix, \ B = \begin{bmatrix} 18 \\ 5 \end{bmatrix}[/tex]

From the equation, A·X = B, we have;

[tex]\therefore X = \dfrac{B}{A} = A^{-1} \times B[/tex]

Where: A⁻¹ is the inverse matrix of A, which is found as follows;

[tex]If\ A = \begin{bmatrix} a&b \\ c&d \end{bmatrix}[/tex]

[tex]Then, \ A^{-1} = \dfrac{1}{a\cdot d-b\cdot c} \cdot \begin{bmatrix} d& -b \\ -c& a \end{bmatrix}[/tex]

Which gives the value of A⁻¹ obtained from the coefficient matrix, A = [tex]\begin{bmatrix} 3&4 \\ 4& -1 \end{bmatrix}[/tex] as follows;

[tex]A^{-1} = \begin{bmatrix} 3 & 4 \\ 4 & - 1 \end{bmatrix}^{ - 1} = \dfrac{1}{(3 \times - 1) - (4 \times 4)} \times \begin{bmatrix} - 1 & - 4 \\ - 4 & 3 \end{bmatrix}[/tex]

[tex]A^{-1} = \dfrac{1}{(3 \times - 1) - (4 \times 4)} \times \begin{bmatrix} - 1 & - 4 \\ - 4 & 3 \end{bmatrix} = \begin{bmatrix} \dfrac{ - 1}{ - 19} & \dfrac{ - 4}{ - 19} \\\\ \dfrac{ - 4}{ - 19} & \dfrac{3}{ - 19} \end{bmatrix}[/tex]

The variable matrix, [tex]X = A^{-1} \times B[/tex], which gives the value of the variables in the solution is therefore;

[tex]X = \begin{bmatrix} x \\ y \end{bmatrix} = \begin{bmatrix} \dfrac{ - 1}{ - 19} &\dfrac{ - 4}{ - 19} \\ \\\dfrac{ - 4}{ - 19} &\dfrac{3}{ - 19} \end{bmatrix} \times \begin{bmatrix} 18 \\ 5 \end{bmatrix} = \begin{bmatrix} 2 \\ 3 \end{bmatrix}[/tex]

[tex]\begin{bmatrix} x \\ y \end{bmatrix} = \begin{bmatrix} 2 \\ 3 \end{bmatrix}[/tex]

Therefore;

x = 2 and y = 3

Learn more about matrices here:

https://brainly.com/question/16886950

#SPJ1

Find the distance between:(4,-9) and(-8,0)Round your answer to the nearest hundredth.

Answers

The distance between 2 points (x1, y1) and (x2, y2) is calculated as:

[tex]\text{distance}=\sqrt{(x_2-x_1)^2+(y_2-y_1)^2}[/tex]

So, if we replace (x1, y1) by (4, -9) and (x2, y2) by (-8,0), we get:

[tex]\begin{gathered} \text{distance}=\sqrt{(-8-4)^2+(0-(-9))^2} \\ \text{distance}=\sqrt{(-12)^2+(9)^2} \\ \text{distance}=\sqrt{144+81} \\ \text{distance}=\sqrt{225} \\ \text{distance}=15 \end{gathered}[/tex]

Answer: the distance is 15

Review: Solve for Area AND Circumference. A giant holiday cookie has a radius of 5 inches. What is the area of the cookie? What is the circumference of the cookie?

Answers

Remember that the formual for the area of a circle is:

[tex]A=\pi r^2[/tex]

And the formula for the circumference is:

[tex]C=2\pi r[/tex]

Using this formulas and the data given,

[tex]\begin{gathered} A=\pi(5^2)\Rightarrow A=78.54 \\ C=2\pi(5)\Rightarrow A=31.42 \end{gathered}[/tex]

The cookie has an area of 78.54 square inches and a circumference of 31.42 inches

Find the length of AB given that DB is a median of the triangle AC is 46

Answers

ANSWER:

The value of AB is 23

STEP-BY-STEP EXPLANATION:

We know that AB is part of AC, and that DB cuts into two equal parts (half) since it is a median, therefore the value of AB would be

[tex]\begin{gathered} AB=\frac{AC}{2} \\ AB=\frac{46}{2} \\ AB=23 \end{gathered}[/tex]

A woman who has recovered from a serious illness begins a diet regimen designed to get her back to a healthy weight. She currently weighs 106 pounds. She hopes each week to multiply her weight by 1.04 each week.

Answers

The required exponential function would be W = 106 × 1.04ⁿ for the weight after n weeks.

What is an exponential function?

An exponential function is defined as a function whose value is a constant raised to the power of an argument is called an exponential function.

It is a relation of the form y = aˣ  in mathematics, where x is the independent variable

The given starting weight for the diet program is 106 pounds. Because the weight is expected to be multiplied by 1.04 pounds each week, the weight will develop exponentially with an initial value of 106 pounds and a growth factor of 1.04 pounds. Then, for the weight after weeks, the exponential function is given by,

W  = W(n) = Pb'

Here P = 106 and b = 1.04

Hence the required formula is,

⇒ W = 106 × 1.04ⁿ

Thus, the required exponential function would be W = 106 × 1.04ⁿ for the weight after n weeks.

Learn more about the algebraic expression here :

brainly.com/question/21751419

#SPJ1

The question seems to be incomplete the correct question would be

A woman who has recovered from a serious illness begins a diet regimen designed to get her back to a healthy weight. She currently weighs 106 pounds. She hopes each week to multiply her weight by 1.04 each week. Then, find the exponential function for the weight after weeks.

Can you help me answer part A and part B?

Answers

Part A.

Given:

P = (5, 4), Q = (7, 3), R = (8, 6), S = (4, 1)

Let's find the component of the vector PQ + 5RS.

To find the component of the vector, we have:

[tex]=\lparen Q_1-P_1,Q_2-P_2)=<7-5,3-4>[/tex]

For vector RS, we have:

[tex]=\lparen S_1-R_1,S_2-R_2)=<4-8,1-6>[/tex]

Hence, to find the vector PQ+5RS, we have:

[tex]\begin{gathered} =<7-5,3-4>+5<4-8,1-6> \\ \\ =\left(2,-1\right)+5\left(-4,-5\right) \\ \\ =\left(2,-1\right)+\left(5\ast-4,5\ast-5\right) \\ \\ =\left(2,-1\right)+\left(-20,-25\right) \end{gathered}[/tex]

Solving further:

[tex]\begin{gathered} =<2-20,-1-25> \\ \\ =<-18,-26> \end{gathered}[/tex]

Therefoee, the component of the vector PQ+5RS is:

<-18, -26>

• Part B.

Let's find the magnitude of the vector PQ+5RS.

To find the magnitude, apply the formula:

[tex]m=\sqrt{\left(x^2+y^2\right?}[/tex]

Thus, we have:

[tex]\begin{gathered} m=\sqrt{\left(-18^2+-26^2\right?} \\ \\ m=\sqrt{324+676} \\ \\ m=\sqrt{1000} \\ \\ m=\sqrt{10\ast10^2} \\ \\ m=10\sqrt{10} \end{gathered}[/tex]

Therefore, the magnitude of the vector is:

[tex]10\sqrt{10}[/tex]

ANSWER:

Part A. <-18, -26>

Part B. 10√10

The question is which of these statements are true about radicals exponents and rational exponents

Answers

We have the following:

I)

[tex]\sqrt[n]{a}=a^{\frac{1}{n}}[/tex]

It´s true

II)

[tex]a^{\frac{1}{2}}=\sqrt[]{a}[/tex]

It´s true

III)

[tex]\begin{gathered} a^{\frac{p}{q}}=\sqrt[p]{a^q}=(\sqrt[p]{a})^q \\ (\sqrt[p]{a})^q=(a^{\frac{1}{p}})^q=a^{\frac{q}{p}} \end{gathered}[/tex]

It´s false

IV)

[tex]\sqrt[]{a}[/tex]

It´s true

V)

[tex]\begin{gathered} a^{\frac{1}{n}}=\sqrt[]{a^n} \\ \sqrt[]{a^n}=a^{\frac{n}{2}} \end{gathered}[/tex]

It´s false

What does the slower car travel at Then what does the faster car travel at

Answers

Given that two cars are 188 miles apart, travelling at different speeds, meet after two hours.

To Determine: The speed of both cars if the faster car is 8 miles per hour faster than the slower car

Solution:

Let the slower car has a speed of S₁ and the faster car has a speed of S₂. If the faster speed is 8 miles per hour faster than the slower car, then,

[tex]S_2=8+S_1====\text{equation 1}[/tex]

It should be noted that the distance traveled is the product of speed and time. Then, the total distance traveled by each of the cars before they met after 2 hours would be

[tex]\begin{gathered} \text{distance}=\text{speed }\times time \\ \text{Distance traveled by the faster car after 2 hours is} \\ =S_2\times2=2S_2 \\ \text{Distance traveled by the slower car after 2 hours is} \\ =S_1\times2=2S_1 \end{gathered}[/tex]

It was given that the distance between the faster and the slower cars is 188 miles. Then, the total distance traveled by the two cars when they meet is 188 miles.

Therefore:

[tex]\begin{gathered} \text{Total distance traveled by the two cars is} \\ 2S_1+2S_2=188====\text{equation 2} \end{gathered}[/tex]

Combining equation 1 and equation 2

[tex]\begin{gathered} S_2=8+S_1====\text{equation 1} \\ 2S_1+2S_2=188====\text{equation 2} \end{gathered}[/tex]

Substitute equation 1 into equation 2

[tex]\begin{gathered} 2S_1+2(8+S_1)=188 \\ 2S_1+16+2S_1=188 \\ 2S_1+2S_1=188-16 \\ 4S_1=172 \end{gathered}[/tex]

Divide through by 4

[tex]\begin{gathered} \frac{4S_1}{4}=\frac{172}{4} \\ S_1=43 \end{gathered}[/tex]

Substitute S₁ in equation 1

[tex]\begin{gathered} S_2=8+S_1 \\ S_2=8+43 \\ S_2=51 \end{gathered}[/tex]

Hence,

The slower car travels at 43 miles per hour(mph), and

The faster car travels as 51 miles per hour(mph)

Evaluate 2(x - 4) + 3x - x^2 for x = 2.O A. -6O B. -2O C. 6O D. 2

Answers

C. 6

Explanation

To evaluate an algebraic expression means to find the value of the expression when the variable is replaced by a given number.so

Step 1

given

[tex]2(x-4)+3x-x^2[/tex]

a)let

[tex]x=2[/tex]

b) now, replace and calculate

[tex]\begin{gathered} 2(x-4)+3x-x^2 \\ 2(2-4)+3(2)-(2^2) \\ 2(-2)+6-4 \\ -4+6-4 \\ -4+6-4=6 \end{gathered}[/tex]

therefore, the answer is

C. 6

I hope this helps you

Julie wants to purchase a jacket that costs $125. So far she has saved $42 and plans tosave an additional $25 per week. She gets paid every Friday, so she only gets money toput aside once a week. How many weeks, x, will it take for her to save at least $125?

Answers

cost of the jacket = $125

money saved = $42

extra savings = $25/week

Ok

125 = 42 + 25w

w = number of weeks

Solve for w

125 - 42 = 25w

83 = 25w

w = 83/25

w = 3.3

She needs to save at least 3.3 weeks

Select the correct answer from the drop-down menu.Find the polynomial,{4'" is the solution set of

Answers

Let P(x) be the polynomial such that the given set is its solution set.

Now notice that:

[tex]\begin{gathered} x=-\frac{1}{3}\Rightarrow x+\frac{1}{3}=0\Rightarrow3x+1=0, \\ x=4\Rightarrow x-4=0. \end{gathered}[/tex]

Therefore (x-4) and (3x+1) divide to P(x), then:

[tex]\begin{gathered} Exists\text{ k such that:} \\ P(x)=k(x-4)(3x+1). \end{gathered}[/tex]

Simplifying the above result we get:

[tex]P(x)=k(3x^2-11x-4).[/tex]

Setting k=1 we get that:

[tex]P(x)=3x^2-11x-4.[/tex]

Answer: Second option.

what is the area if one of the triangular side of the figure?

Answers

Compound Shape

The shape of the figure attached consists on four triangles and one square.

The base of each triangle is B=12 cm and the height is H=10 cm, thus the area is:

[tex]A_t=\frac{BH}{2}[/tex]

Calculating:

[tex]A_t=\frac{12\cdot10}{2}=60[/tex]

The area of each triangle is 60 square cm.

Now for the square of a side length of L=12.

The area of a square of side length a, is:

[tex]A_s=a^2[/tex]

Calculate the area of the square:

[tex]A_s=12^2=144[/tex]

The total surface area is:

A = 60*4 + 144

A= 240 + 144

A = 384 square cm

Which best represents the number of square centimeters in a square foot?A 366 square centimeters B 144 square centimeters C 930 square centimeters D 61 square centimeters

Answers

Answer:

C. 930 square centimeters

Explanation:

First, recall the standard conversion between cm and ft.

[tex]1\text{ ft}=30.48\operatorname{cm}[/tex]

Therefore:

[tex]\begin{gathered} (1\times1)ft^2=(30.48\times30.48)cm^2 \\ =929.03\operatorname{cm}^2 \\ \approx930\text{ square cm} \end{gathered}[/tex]

The correct choice is C.

Four points are labeled on the number line. M K L zo 0.5 1 Which point best represents 3? F. Point K G. H. Point 2 Point M Point N J.

Answers

The point that best represents 1/3 is point M .

The number line ranges from 0 to 0.5 with 10 divi

In a probability experiment, Craig rolled a six-sided die 62 times. The die landed on a number greater than three 36 times. What is the ratio of rolls greater than three to rolls less than or equal to three?

Answers

Answer:

31/55

Step-by-step explanation:

A bus travels 8.4 miles eastand then 14.7 miles north.What is the angle of the bus'resultant vector?Hint: Draw a vector diagram.O[?]

Answers

A bus travels 8.4 miles east and then 14.7 miles north.

What is the angle of the bus resultant vector?

see the figure below to better understand the problem

The angle of the bus resultant vector R is equal to

tan(x)=14.7/8.4

mm

In the diagram below, BS and ER intersect as show. Determine the measure of

Answers

[tex]\begin{gathered} \angle RAS=\angle BAE\text{ (Vertically opposite angles are equal)} \\ \angle RAS=(9x+24)^0 \end{gathered}[/tex][tex]\angle BAR+\angle RAS=180^0(sum\text{ of angles on a straight line)}[/tex][tex]\begin{gathered} 11x+16+9x+24=180 \\ 11x+9x+16+24=180 \\ 20x+40=180 \\ 20x=180-40 \\ 20x=140 \\ x=\frac{140}{20} \\ x=7^0 \end{gathered}[/tex][tex]\begin{gathered} \angle RAS=9x+24 \\ \angle RAS=9(7)+24 \\ \angle RAS=63+24 \\ \angle RAS=87^0 \end{gathered}[/tex]

Find the value of 4v-8 given that 11v-8 = 3.Simplify your answer as much as possible.

Answers

Given that 11v - 8 = 3

add 8 to both sides

11v = 3 + 8

11v = 11

Divide both sides by 11

v = 1

to simplify 4v - 8

= 4 (1) - 8

= 4 - 8

= -4

Answer:

-4

Explanation
Other Questions
Maria is planting a row of flowers in a bed 77 feet long. The instructions say to space the plants 1 foot apart, allowing room for 77 flowers. The flowers come in flats containing 6 plants per flat. A square pyramid has a volume of 108 cubic feet and a height of 4 feet.What is the length of each side of the base of the pyramid?A 4 ftOLOB. 9 ftC. 18 ftD. 27 ftO E. 81 ftHelp please very hard Read it please please 2 Estelle is having her birthday party at Gold Frames Art Museum this year. A party packagecosts $265 and covers the entrance fee for guests, a private party room, and an activityguide. Estelle upgraded her package to include a favor for each of her 8 guests, bringing thetotal to $305.Which equation can you use to find f, the cost of each favor?265(f+ 8) = 305265f + 8 = 305How much did each favor cost?Submit8f +265 3058(f+265): = 305 7/5 convert improper fractions Factor the polynomial completely if possible. If the expression cannot be factored, enter the expression as is total quality management emphasizes a. a system where strong managers are the only decision makers b. the responsibility of the quality control staff to identify and solve all quality-related problems. c. a process where mostly statisticians get involved d. a commitment to quality that goes beyond internal company issues to suppliers and customers. answer this, please?Sidney made root beer floats for her friends when they came over. The table shows the ratio of cups of ice cream to cups of soda used to make the floats.Ice Cream (cups) Soda (cups)3.5 10.58 2412.5 37.519 ?At this rate, how much soda will Sidney use for 19 cups of ice cream? 30 cups 38 cups 57 cups 72 cups HELP ME PLEASE ASAP!!!2. Use the space provide to respond to each statement concerning the given graph of a radical function.a.) State the domain of the function. b.) State the range of the function.c.) Identify the end behavior of the function.d.) Identify the y intercept. Write as an ordered pair.e.) Explain how you know this function is increasing? g a stock has never before paid a dividend but is expected to make its first dividend payment in 5 years. if the first dividend payment is expected to be $3 and dividends are expected to grow at 6%, what is the stock expected to be worth four years from now if investors require a 12% return? Greg purchased furniture for his home. He purchased a chair for $320 and a lamp for $55. The store charges 7% sales tax. How much did Greg pay in tax? Why did the sale of indulgences become a critical point of focus during the renaissance but not during the middle ages?. 3. Given the perimeter of the figure is 76 ft, Findthe dimensions of the shape.(4x-5) ft(25-x) ft(13 + x) ftT(3x + 1) ft Using everyday knowledge, indicate whether the if-then statements are correct forward-only or both forward and reverse.Statement 1: If Bob is Sallys spouse, then Sally is Bobs spouse.Statement 2: If the light is red Northbound, then the traffic is stopped. Rewrite the function by completing the square. f (x)= x^2 - 9x + 14f (x) = _ ( x + _ )^2 + _ solon's reforms in athens select one: a. established democracy in athens. b. established social equality. c. gave citizens a place in the assembly. d. abolished slavery. e. eliminated the position of chief magistrate. Write a quadratic equation in standard form with the given roots. a. Write a quadratic equation with a double root of -5. how is [tex] {1}^{4} [/tex]differ from [tex] { {0.1}^{4} }^{} [/tex] The Muffin Shop makes no-fat blueberry muffins that cost $.70 each. The Muffin Shop knows that 15% of the muffins will spoil. If The Muffin Shop wants 40% markup on cost and produces 800 muffins, what should The Muffin Shop price each muffin? 3. Jackson's soccer coach filled the teams' water container with 40 quarts ofwater. Since 32 ounces equal 1 quart, how many times can a soccer player fill a16-ounce water bottle before using all the water?